Problem on triangle and square

Geometry Level 3

As shown in the diagram above, A B C ABC is a right-angled triangle with A B = 3 AB=3 , A C = 4 AC=4 and C A B = 9 0 \angle CAB=90 ^{\circ} . The point F F is the centre of the square B C D E BCDE .

Suppose x x is the length of the segment A F AF . Find the value of 1000 x , \displaystyle\left\lfloor 1000x \right\rfloor, where \lfloor \cdot \rfloor denotes the floor function .


The answer is 4949.

This section requires Javascript.
You are seeing this because something didn't load right. We suggest you, (a) try refreshing the page, (b) enabling javascript if it is disabled on your browser and, finally, (c) loading the non-javascript version of this page . We're sorry about the hassle.

3 solutions

Chan Lye Lee
Nov 13, 2018

Add another 3 right-angled triangles, as shown. Then we see that A F = 1 2 A H = 1 2 × 7 2 4.9497 AF=\frac{1}{2}AH =\frac{1}{2} \times 7\sqrt{2}\approx 4.9497 . Hence 1000 x = 4949 \displaystyle\left\lfloor 1000x \right\rfloor= \boxed{4949} .

It's NOT 1/2 x SQRT(2) x SQRT(7). Rather, it's 1/2 x SQRT(2) x 7.

Why Portant - 2 years, 7 months ago

Log in to reply

@Why Portant Edited, thanks.

Chan Lye Lee - 2 years, 7 months ago

Since quadrilateral A B F C ABFC is cyclic, we have by Ptolemy's theorem in cyclic quad. ABFC

A F × B C = A B × C F + A C × B F AF\times BC = AB \times CF + AC \times BF

A F × 5 = 3 × 5 2 + 4 × 5 2 \Rightarrow AF \times 5 = 3 \times \dfrac{5}{\sqrt{2}} + 4 \times \dfrac{5}{\sqrt{2}}

A F = 7 2 4.949. \therefore AF = \dfrac{7}{\sqrt2} \approx4.949.

Q . E . D \boxed{Q.E.D}

I solved it the same way you did.

Abu Nazam Sk Md Tamimuddin - 2 years, 5 months ago

Since, A B C ABC and. B C F BCF are right triangles and share common hypotenuse B C , BC, they can be inscribed in a Circle as shown. Let, A = ( a , b ) , B = ( B C 2 , 0 ) , C = ( B C 2 , 0 ) , F = ( 0 , B C 2 ) A= (-a, -b) , B= (-\dfrac {BC}{2} , 0) , C =(\dfrac {BC}{2},0) , F = (0, \dfrac {BC}{2})

A A will lie in 3 r d 3^{rd} quadrant if you plot this on a Cartesian plane. B C BC will lie along X-axis and F M FM along Y-axis. M M will be at Origin.

A r ( A B C ) = 1 2 × A B × A C = 1 2 × B C × A N Ar(ABC) = \dfrac {1}{2}\times AB \times AC = \dfrac {1}{2}\times BC \times AN

A C = 4 , A B = 3 , B C = A B 2 + A C 2 = 5 AC = 4 , AB = 3 , BC = \sqrt{AB^{2} + AC^{2}} = 5

We get, A N = 2.4 = b AN = 2.4 = b

Since, A A lies on Circle it can be parametrically written as ( a , b ) = ( 2.5 cos β , 2.5 sin β ) (-a, -b) = (-2.5\cosβ,-2.5\sinβ)

Therefore, sin β = 2.4 2.5 cos β = 1 sin 2 β = 0.7 2.5 \sinβ = \dfrac {2.4}{2.5} \Rightarrow \cos β = \sqrt{1 - \sin^{2}β} = \dfrac {0.7}{2.5}

a = 0.7 \Rightarrow a = 0.7

A = ( 0.7 , 2.4 ) A = (-0.7 , -2.4) and F = ( 0 , 2.5 ) F = (0 , 2.5)

So, A F = ( 0.7 0 ) 2 + ( 2.4 2.5 ) 2 = 4.94974746831 AF = \sqrt{(0.7 - 0)^{2} + (-2.4 - 2.5)^{2}}=\boxed{ 4.94974746831}

Finally we get , A N S W E R : 4949 ANSWER:\boxed{4949}

0 pending reports

×

Problem Loading...

Note Loading...

Set Loading...